How to find the total displacement of an object ?

Answers

Answer 1

Answer:

it can be calculated by measuring the final distance away from a point, and then subtracting the initial distance


Related Questions

As an IT technician for your company, you have been notified that the Windows domain does not seem to be functioning properly. Being familiar with domains, you are fairly confident you know what the issue is. But just to be safe, you take the applicable time to gather additional information and to identify what, if anything, has changed.

Which of the following is the BEST next step?

A. Determine the appropriate fix.
B. Create a hypothesis.
C. Implement the fix.
D. Identify what has changed.
E. Gather information.

Answers

Answer: Create a hypothesis

Explanation:

From the information given, information has been gathered and the identification to ascertain if there's a change. Then, an hypothesis has to be created in order to know what the problem is.

One has to carry out some research in order to know what went wrong and should also validate the hypothesis by consulting with ones peers. By doing this, the most likely causes of the issues will be gotten.

5. What type of chemical reaction is shown?
1 point
2 AgNO3 + CuCl2
-
2 AgCl + 2 Cu(NO3)2
Synthesis
Decomposition
Singe Replacement
Double Replacement
Combustion

Answers

Answer:

double Replacement

Explanation:

The mutual replacement of radical and ions between the two compounds or molecules is called double replacement.

General equation : AB+CD = AD +B

Q) A farmer moves along the boundary of a
square field of side 10 min 40 s .What will
be the magnitude of displacement of the
farmer at the end of 2 minutes 20 seconds
from his initial position?​

Answers

The answer will be 8 because kedks

When the surface of a metal is exposed to blue light, electrons are emitted. If the intensity of the blue light is increased (so the light is just brighter), what value will also increase?
A. the maximum kinetic energy of the ejected electrons
B. the number of electrons ejected per second
C. the time lag between the onset of the absorption of light and the ejection of electrons

please help!!!

Answers

So, we know that this question is about photoelectric effect.

What is photoelectric effect?

-Photoelectric effect is a phenomenon where electrons are released from a material whenever an electromagnetic radiation such as light, hits it.

-This phenomenon is first discovered by Heinrich Rudolf Hertz in 1887 whereas Albert Einstein, in 1905, resolved the conflict between the photoelectric effect and Maxwell's electromagnetic wave theory of light.

Now to your question, the intensity of light only determines the number of the electrons released, not their maximum kinetic energy because this are proportional to the light's frequency.

Also, there is no time lag detected between the entry of radiation and the emission of electrons in experiments held.

So, with letter A and C out, we can therefore conclude that the answer for this is Letter B

The correct answer is B: the number of electrons ejected per second

The Einstein photoelectric equation states that;

[tex]KE = hf - hfo[/tex]

Where

KE = kinetic energy of the ejected photo electrons

h = Plank's constant

f = frequency of incident light

fo = threshold frequency.

The kinetic energy of ejected photoelectrons depends on frequency of incident photon and not on the intensity of the incident photon.

However, the number of photoelectrons ejected per second depends on the intensity of the incident blue light.

https://brainly.com/question/9260704

2. CaCl2 (s) + 2H20 ---> Ca(OH)2 (aq) + 2HCl (g) + heat
Is it an endothermic reaction?

Answers

Answer:

no

Explanation:

Calcium chloride is a chemical compound made up of calcium ions and chlorine ions. ... Mixing calcium chloride with water is an exothermic reaction, which means that the combination of the two substances releases heat. Thus, when you add calcium chloride to water, the solution heats.

Convergent boundaries are responsible for

Answers

Answer:

A

Explanation:

The plate collisions that occur in these areas can produce earthquakes, volcanic activity, and crustal deformation.

hope it helps you

hey everyone


what is optics??​

Answers

Explanation:

Optics is a branch of physics that is the study of light and vision. ... The branch of physics dealing with the nature and properties of electromagnetic energy in the light spectrum and the phenomena of vision. In the broadest sense, optics deals with infrared light, visible light, and ultraviolet light.

A 45000 watt crane operating at full power lifts a 2100 kg object vertically for 17.4 seconds. How high has the crane lifted the object?

Answers

Answer:

Explanation:

We need the power equation here, which is:

Power = (F * Δx)/time

where F * Δx is the amount of work done.

F is a force which is measured in Newtons. We are given the mass of the object, but since we need a Force measure, we need the weight of the object:

F = 2100(9.0)

F = 21000 to the correct number of sig dig.

Now we can plug in the values we have and solve for the displacement, Δx:

[tex]45000=\frac{21000x}{17.4}[/tex] and isolating x:

[tex]\frac{17.4(45000)}{21000}=x[/tex] so

x = 37 m

Using a 100 year time frame,what is the CO2e of 1 tonne of CO2,2 tonne or methane,and 3 tonnes of nitrous oxide

Answers

Well, let's see what we've got here:

                    ( 4⁻³ · 3⁴ · 4² )  /  ( 3⁵ · 4⁻² ) .

The two simple rules of exponents that we're

going to use here are:

-- Two multiply two numbers with the same base,

add their exponents.    Like   (A³) · (A²) = A⁵ .

But the bases have to be the same.

-- A negative power just means it belongs in the

other section of a fraction.

  A negative power on top means it belongs on the bottom.

  A negative power on the bottom means it belongs on top.

  Like    A⁻²  means  1/A² .  And  1/B⁻³  means  B³ .

That's all you need in order to clean up the big fraction

in the question.  But in order to see where you can use

these rules, you need to re-arrange things first.

Original:                          ( 4⁻³ · 3⁴ · 4² )  /  ( 3⁵ · 4⁻² )

Let's send the  4⁻³

to the bottom

where it belongs:              (   --  3⁴ · 4² )  /  4³ · ( 3⁵ · 4⁻² )

Not take that  4⁻² from

the bottom, and put it on

top, where it belongs:       (  4² · 3⁴ · 4² )  /  4³ · ( 3⁵    -- )

Multiply the 4²s on top:     (  4⁴ · 3⁴       )  /  ( 4³ · 3⁵ )

Now let me break this up.

Not changing anything, just

writing it in a different way:      ( 4⁴  /  4³ ) · ( 3⁴ / 3⁵ )

Look at the first fraction:  4⁴ / 4³ .

Divide top and bottom by  4³ , and it becomes just  4 .

Now look at the second fraction:  3⁴ / 3⁵ .

Divide top and bottom by  3⁴ , and it becomes just  1/3 .

So in the end, we're left with just      4 / 3 .

And THAT is exactly equal to the original big messy fraction

in the question.  It has exactly the same numerical value, but

you'd never know it when you see it, because it's a lot simpler.

There are a lot of other ways we could have manipulated and

massaged the original fraction, but the steps are the same:

-- Multiply numbers with the same base, by adding the exponents.

-- Remember that a number with a negative exponent belongs

in the other section of the fraction, with a positive exponent.

If the light ray hits the first mirror and continues to bounce off the other mirrors inside the box, determine where the light ray will eventually hit on the border

Answers

Answer: Light ray X will eventually hit the borader

Explanation:

Please help me :))

Which golf ball would hit the surface with the greatest impact: a ball that fell one meter above the surface of the Earth or a ball that fell one meter above the surface of the Moon?

Answers

Answer:

the one above the surface of earth

Explanation:

earth has gravity the ball of the moon would float away

electromotive force in a circuit;
A) causes free electrons to flow
B) increases the circuit resistance
C) maintains circuit resistance
D) it needed to make the circuit complete

Answers

Answer:

A causes free electrons to flow

Explanation:

The amount of force that causes electrons to flow in a conductor is called electromotive force.

A trip is taken that passes through the following points in order
Point A Om
Point B 15.0 m
Point C -30.0 m
Point D 20.0 m
Point E -10.0 m
Point F 5.0 m
What is the displacement from Point B to Point E?

A) 5.0 m right
B) 85 m right
C) 25.0 m left
D) 85 m left

Answers

Answer:

The displacement from point B to point E is 25.0 m left

Đặt vào đầu hai dây dẫn cùng một hiệu điện thế bằng nhau U,có điện trở lần lượt là R1 và R2.Ta thấy cường độ dòng điện qua dây dẫn thứ nhất I1 lớn gấp hai lần cường độ dòng điện qua dây dẫn thứ hai I2

Answers

Answer:

The resistance ratio of first conductor to the second conductor is 1 : 2.

Explanation:

Current in first conductor = I1

Current in second conductor = I2 , I1 = 2 I2

According to the Ohm's law

[tex]V= I_1R_1..... (1)\\V = I_2 R_2 .... (2)\\[/tex]

From (1) and (2)

[tex]I_1 R_1 = I_2 R_2\\\\2I_2 R_1 = I_2 R_2\\\\2 R_1 = R_2[/tex]


A 40kg girl and a 50kg boy are facing each other on a friction-free rollerskates. The girl pushes the boy, who moves away at a speed of 3 m/s. What is the girl's
speed?
Your answer

Answers

Answer:

Explanation:

The Law of Momentum Conservation, just like the Law of Thermodynamics about energy, says that momentum is neither created nor destroyed but is conserved, meaning it has to go somewhere. If the girl pushes the boy and they are both on friction free skates, then the girl will also react to the push. Momentum Conservation says

[tex](m_gv_g+m_bv_b)_b=(m_gv_g+m_bv_b)_a[/tex] In words this says that the mass times the velocity of the girl plus the mass times the velocity of the boy before the push has to equal the mass times velocity of the girl plus the mass times velocity of the boy after the push. Mathematically,

[tex](40.0*0+50.0*0)_b=(40.0v+50.0*-3)_a[/tex]

The left side of this is equal to 0. On the right, I made the velocity of the boy negative. We could have made it positive and it wouldn't have mattered. The sign will only be important to the result because if the sign of the girl's velocity is the same as the boy's, she is moving in the same direction as he is; if it's different, she is moving in the opposite direction.

0 = 40.0v - 150.0 and

-40.0v = -150.0 so

v = 3.75 This means that when she pushes the boy one way, mometum is conserved and she moves in the opposite direction and at a greater velocity (because her mass is less). Physics is a wonderful thing, isn't it!?

5. A wire, resistance 23.6 carries a current. 5.9 A. What is the
potential difference across the wire?
a. V = 139 V
c. V=4V
b. V = 17.7 V
d. V = 0.25 V

Answers

Answer:

d

Explanation:

Answer:

B

Explanation:

What's the meaning of longitudinal?

Answers

Answer:

A longitudinal study is a research design that involves repeated observations of the same variables over short or long periods of time. It is often a type of observational study, although they can also be structured as longitudinal randomized experiments.

Explanation:

I am confused and need help with the question above??

Answers

It’s b it’s going clockwise

HELP PLSSS I HAVE AN EXAM MONDAY AND I THINK THIS IS GONNA BE ON ITTTT
A fish is at the western bank of a river that is 64 m wide and has a current with a velocity of 0.90 m/s [S]. The fish swims directly across the river going due east. The fish can swim at a speed of 0.2 m/s.

a) How long does it take for the fish to get across the river
b) What is the resulting velocity of the fish
c) When the fish arrives on the opposite bank, how far is it from being at the point directly across from where it started?

The answers are here, please show work on how to get these
a) 3.2 x 10^2
b) 0.9 m/s (S 13 E)
c) 2.9 x 10^2

Answers

Answer:

(a) 3.2 x 10²s

(b) 0.9 m/s (S 13 E)

(c) 2.9 x 10²m

Explanation:

The sketch illustrating the scenario has been attached to this response.

As shown;

The fish swims due east with a velocity [tex]V_{x}[/tex] = 0.2m/s

The river current has a velocity [tex]V_{y}[/tex] due South = 0.9m/s

The resultant of the velocity is V

The width of the river is x = 64m

(a) To calculate how long it took the fish to get across the river, we know that velocity is the rate of change in distance, therefore we can use the relation;

V = [tex]\frac{d}{t}[/tex]      -------------(i)

Where;

V = velocity of the fish = [tex]V_{x}[/tex] = 0.2m/s

d = distance from the start to the end = width of the river = x = 64m

t = time taken to move for that distance

Make t subject of the formula in equation (i);

t = [tex]\frac{d}{V}[/tex]

Substitute the values of d and V into the equation;

t = [tex]\frac{64m}{0.2m/s}[/tex]

t = 320 s

t = 3.20 x 10²s

Therefore, the time taken for the fish to get across the river is 3.20 x 10²s

(b) The resulting vector of the fish is V whose magnitude is the algebraic sum of vectors  [tex]V_{x}[/tex] and  [tex]V_{y}[/tex], and direction is given by θ. i.e

The magnitude of the resulting vector is;

|V| = [tex]\sqrt{(V_x)^2 + (V_y)^2}[/tex]

|V| = [tex]\sqrt{(0.2)^2 + (0.9)^2}[/tex]

|V| = [tex]\sqrt{(0.04) + (0.81)}[/tex]

|V| = [tex]\sqrt{(0.85)}[/tex]

|V| = 0.92m/s

|V| ≅ 0.9m/s

The direction of the resulting vector θ and is given by;

tan θ = [tex]\frac{V_y}{V_x}[/tex]

tan θ = [tex]\frac{0.9}{0.2}[/tex]

tan θ = 4.5

θ = tan⁻¹ ( 4.5)

θ = 77.47° South of East.

θ  ≈ 77.5° South of East.

Subtracting θ = 77.5° from 90° gives its value East of South

i.e

90 - 77.5 = 12.5° East of South

This can also be written as S12.5°E

Approximating to the nearest whole number gives S 13 E

Therefore, the resulting velocity of the fish is 0.9m/s in the direction S13°E

(c) When the fish arrives on the opposite bank, its distance from being at the point directly across from where it started is the product of the velocity of the river current and the time taken by the fish to get across the river. This point is equivalent to k as shown in the diagram.

Therefore;

distance = velocity of river current x time taken

distance = 0.9m/s x 3.20 x 10²s

distance = 2.88 x 10²m

distance ≅ 2.9 x 10²m

Notice that the velocity of the river current is used since that's the velocity of the fish on the y-axis.

The process of recovering valuable or useful materials from waste or scrap is called
a.
recycling.
c.
rebuilding.
b.
renewing.
d.
reducing.


Please select the best answer from the choices provided

A
B
C
D

Answers

The answer is A) Recycling. Recycling has to do with finding something valuable amongst trash. For example, in a dumpster, someone may just find a nice painting. Mark as brainliest :)

Answer:

hi

Explanation:

According to Archimedes' principle, when will an object float in water?
A. When its volume is less than the volume of water it displaces
B. When its weight is equal to the buoyant force exerted by the water
C. When its density is greater than the density of water
D. When its weight is greater than the weight of water it displaces

Answers

C. When it’s density is greater than the density of water.

Answer:

Its B

Explanation:

a p e x

The two waves shown here are the same distance from point X and are traveling toward each other.

An illustration with a crest 2.0 centimeters high traveling to the right and a second 2.0 centimeters trough traveling toward the left toward point X in the material.
What is the amplitude of the resulting wave when the two waves meet at point X?

0.0 cm
1.0 cm
2.0 cm
4.0 cm

Answers

Answer:

(C) on edge.

Explanation:

Answer:

2.0

Explanation:

trust the process.

Draw Lewis diagram of an oxygen difluoride molecule​

Answers

Explanation:

that is my answer thank me later

Answer:

oxygen difluoride a polar molecule

Convert 162 km/h into m/s.

Answers

Answer:

45 m/s

Explanation:

162 / 3.6 = 45 m/s

Divide by 3.6 to convert km/h to m/s.

Dos carritos se colocan en una mesa sobre la que pueden moverse sin rozamiento y se sujetan comprimiendo un resorte entre ellos. El carrito 1 tiene doble masa que el 2. De pronto se sueltan. A) Compara las velocidades, cantidades de movimiento y energías cinéticas que adquieren, b) ¿Cómo serían las respuestas si la masa del carrito 1 fuese mucho mayor que la del 2?

Answers

Answer:

a)  v₁ = - ½ v₂,  K₁ / K ₂ = ½,)

Explanation:

A) Let's use the conservation of the moment, for this we define the system formed by the spring and the two cars, so the force during the separation is internal, therefore the moment is conserved

initial instant. Before releasing the carts

         p₀ = 0

final instant. After jumping the cars

         p_f = M v₁ + m v₂₂

how the moment was preserved

        p₀ = p_f

        0 = M v₁ + m v₂

        v₁ = - m / M v₂

indicate that M = 2m

        v₁ = - ½ v₂

the kinetic energy of each car is

        K₁ = ½ M v₁²

        K₁ = ½ 2m (v₂/ 2) ²

        K₁ = m ₂v₂² / 4

        K₂ = ½ m v₂²

the relationship between the kinetic energies is

         K₁ / K₂ = ½

B) If the much greater than the mass of car 1 is mass of car 2

        v = - m / M v₂

         In this case, the speed of car 1 is very small, so the car 1 practitioner does not backtrack and car 2 leaves with a lot of speed.

The energy is still conserved, where almost all the energy has it is car 2

A man can jump 2 m on the surface of the earth, calculate the height that he can jump on the surface of the moon?​

Answers

Answer:

hi

Explanation:

it is probably 12 m on the surface of the moon

have a good day

I hope this is help full to you


If you want to decrease the current created by a generator what can you do?

Answers

Answer:

To decrease the current;

1) Use fewer loops or number of turns

2) Use a lower speed of rotation of the coil in the magnetic field

3) Use a weaker magnetic

Explanation:

According to Faraday's Law of induction, which is the basis of the electromagnetism, electromagnetic induction and therefore the basis of the  electric generator, can be written as follows;

[tex]\epsilon = -N \cdot \dfrac{\Delta \phi}{\Delta t}[/tex]

Where;

ε = The induced voltage

N = The number of turns (loops)

ΔФ = The change in the magnetic flux

Δt = The change in the time (the duration)

Given that voltage is directly related to the current, decreasing the voltage, decreases the current

To decrease the voltage, and therefore, the current we can;

1) Reduce the number of loops in the coil

2) Increase the time change per unit change in flux by slowing down the speed of rotation of the generator

3) Decrease the amount of change in the magnetic field per turn, by using a weaker magnetic

Which state has the most fixed shape?
O A. Gas
O B. Solid
O C. Liquid
O D. Plasma

Answers

Answer: Liquid

“A substance will take on the shape of an open container if it is a Liquid. Explanation: The major state of matter are solid, liquid and gas. Liquid usually have a definite volume.”

Answer:

solid

Explanation:

Solids are fixed and solid.

need help with this!

just scienceeeeeee

Answers

Answer:

7)  λ = 0.5 m,  8)  f = 4.8 10¹⁴ Hz

Explanation:

The speed of an electromagnetic wave is

          c = λ f

where c is the speed of light in vacuum c = 3 10⁸ m / s

7) indicate the frequency f = 6.0 10⁸ Hz

we do not know the wavelength

         λ = c / f

       

we calculate

        λ = 3 10⁸ / 6.0 10⁸

        λ = 0.5 m

8) indicate the wavelength  λ = 6.25 10-7 m

we do not know the frequency

         f = c / λ

we calculate

        f = 3 10⁸ / 6.25 10⁻⁷

        f = 0.48 10¹⁵ Hz

        f = 4.8 10¹⁴ Hz

Select the correct answer
If the resistance in a circuit remains constant, what happens to the electric power when the current increases?
OA. The power will increase.
OB.
The power will decrease,
OC. There will be no power.
OD
The current does not affect the power,
Reset
Next

Answers

Answer :

[tex]\large\mathrm{A. Power\:\: will \:\:Increase }[/tex]

Explanation :

We know that,

[tex]\large \mathrm{ \boxed{P = I²R}}[/tex]

where,

P = electric powerR = resistanceI = electric current

And when resistance is constant, the power of the circuit will change proportionally with the change in electric current. therefore, if current increases then the electric power of the circuit will increase too.

[tex]\large\mathfrak{{\pmb{\underline{\orange{hope \: \: i t \: \: helps \: \: you}}{\orange{.....}}}}}[/tex]

Other Questions
Answer please fast Do it now please, come on do it come on If a rod attached to the approaching charge if the rod consists of "stiff" spring-like bonds for which atoms undergo small oscillations. What can we say, about these springlike bonds when the charge is first, furthest away and second, closest to the source charge In circle P, BC = 24 ft. What is the length of arc AC?A)B)2C)4D)8 Solve x2+y3 = 1 for x. what was ideology as a factor in pakistan foreign policy 58. Which of the following Kingdoms contain multicellular organisms? * I need the answer asap I need it to pass summer school.Drag the tiles to the correct boxes to complete the pairs. Match the lines from Lewis Carroll's "Jabberwocky" to the sound effects used in them. 1.He took his vorpal sword in hand; Long time the manxome foe he sought- 2.Beware the Jabberwock, my son! The jaws that bite, the claws that catch! 3.One, two! One, two! And though and through The vorpal blade went snicker-snack! 4.He left it dead, and with its head He went galumphing back. 5.Come to my arms, my beamish boy! O frabjous day! Callooh! Callay!" alliteration _>assonance _>consonance _>repetition _>internal rhyme _> A system of equation is shown y=-2x+by= 3x+bWhat value of b is required so that the system has the solution (0,6)b= Please no link I need help badly! Solve for x.2/3 (x 7) = 2 all first level consumers are carnivore: True or false? What country have a sand If g(x)=x2 - 5 and 1(x)=7x-11, then what is the value of h(g(3)) ? Plz help, don't put link, and don't answer if you DON'T know[tex]\pi \\[/tex] as 3The volume would be...The surface area would be.... trong tc phm' ch ngha x hi t khng tng n khoa hc'. ph.nghen khi ut khi niemj ca ch ngha x hi khoa hc:' nghin cu nhng iu kin lch s v do , nghin cu ngay bn cht ca s bin i y v bng cch y lm cho giai cp hin nay ang b p bc v c x mnh hon thnh s nghip y hiu r c ngng iu kin, v bn cht s nhgieepj ca chnh h- o s l nhim v ca ch ngha x hi hc, s th hin v mt l lun ca phong tro cng nhn Jefferson Company purchased equipment on January 1, 2017. The equipment cost $60,000 and has an estimated life of 8 years and a salvage value of $8,000. What was the depreciation expense for the equipment for 2018 under the double-declining-balance method? How would write 0.5, 0.65, 2.35, and 1.06 in expanded forms Find the volume. Whoever answers I will reward the brainliest please help Solve for n.56 = -4nA: 224B: -14C: 60D: -4 2. When we lift a stool, its ____________A) gravitational potential energy is increasedB) gravitational potential energy is decreasedC) kinetic energy is decreasedD) gravitational potential energy remains unchanged a circular tabletop has a diameter of 2.1 m. what is its area